Quick parametric equation question

  • Thread starter Thread starter fball558
  • Start date Start date
  • Tags Tags
    Parametric
Click For Summary
To find a parametric representation for the lower half of the ellipsoid defined by the equation 3x^2 + 5y^2 + z^2 = 1, the correct approach involves recognizing that z must be negative for the lower half. The initial attempt to express z as z = sqrt(1 - 3u^2 - 5v^2) was incorrect because it did not account for the sign. A suggested alternative is to use modified spherical coordinates, where x and y are expressed in terms of angles θ and φ, leading to z = cos(φ) with the appropriate bounds for φ to ensure it represents the lower half. This method leverages the symmetry of the ellipsoid for a more straightforward solution.
fball558
Messages
143
Reaction score
0

Homework Statement




Find a parametric representation for the lower half of the ellipsoid 3x^2 + 5y^2 + z^2 = 1
x=u
y=v

z=??

we need to find what z is


The Attempt at a Solution



i solved the equation for z getting

z= sqrt(1-3x^2-5y^2)

then i plugged the given x=u and y=v into equation
to get
z= sqrt(1-3u^2-5v^2)

but that is wrong?
what should i do instead??
 
Physics news on Phys.org
fball558 said:

Homework Statement




Find a parametric representation for the lower half of the ellipsoid 3x^2 + 5y^2 + z^2 = 1
x=u
y=v

z=??

we need to find what z is


The Attempt at a Solution



i solved the equation for z getting

z= sqrt(1-3x^2-5y^2)

then i plugged the given x=u and y=v into equation
to get
z= sqrt(1-3u^2-5v^2)

but that is wrong?
what should i do instead??

shouldn't z be negtive for the lower half?
 
:( yes... i need to learn how to read.
thanks a lot lanedance
that is right :)
 
Did the problem specifically say that you must use x and y themselves as parameters? There is enough "symmetry" here that I would have use "modified" spherical coordinates:
x= \frac{\sqrt{3}}{3}cos(\theta)sin(\phi)<br /> y= \frac{\sqrt{5}}{5}sin(\theta)sin(\phi)<br /> z= cos(\phi)<br /> with 0\le \theta< 2\pi and \pi/2 \le \phi \le \pi.
 
Question: A clock's minute hand has length 4 and its hour hand has length 3. What is the distance between the tips at the moment when it is increasing most rapidly?(Putnam Exam Question) Answer: Making assumption that both the hands moves at constant angular velocities, the answer is ## \sqrt{7} .## But don't you think this assumption is somewhat doubtful and wrong?

Similar threads

  • · Replies 11 ·
Replies
11
Views
3K
Replies
1
Views
1K
  • · Replies 10 ·
Replies
10
Views
2K
  • · Replies 11 ·
Replies
11
Views
3K
  • · Replies 8 ·
Replies
8
Views
2K
  • · Replies 4 ·
Replies
4
Views
3K
  • · Replies 2 ·
Replies
2
Views
2K
Replies
2
Views
2K
  • · Replies 4 ·
Replies
4
Views
2K
Replies
6
Views
2K